How Can We Determine ∂z/∂r for a Complex Multivariable Equation?

  • Thread starter Thread starter catch22
  • Start date Start date
Click For Summary
To find ∂z/∂r for the given multivariable equation, the chain rule is applied, resulting in a lengthy expression involving partial derivatives of z with respect to x and y, and derivatives of x, y, w, and t with respect to r. The calculations yield a complex formula that combines these derivatives. There is a discussion about whether the final answer should be expressed in terms of r or if it is acceptable to leave it in terms of other variables. It is suggested that expressing the answer in terms of u and r may align better with conventional expectations. Ultimately, clarity on the preferred variable expression may depend on the instructor's requirements.
catch22
Messages
62
Reaction score
0

Homework Statement



If z= x2 - y2 + xy
x = w + 4t
y = w2 -5t + 4
w = r2 - 5u
t = 3r + 5u

find ∂z/∂r

Homework Equations

The Attempt at a Solution


This is going to be quite long and tedious so bare with me, but here is my work.
drawing a diagram to help out a bit.

upload_2015-11-6_2-33-2.png
∂z/∂r = (∂z/∂x)(∂x/∂w)(∂w/∂r) + (∂z/∂x)(∂x/∂t)(∂t/∂r) + (∂z/∂y)(∂y/∂w)(∂w/∂r) + (∂z/∂y)(∂y/∂t)(∂t/∂r)

=(2x-y)(1)(2r) + (2x-y)(4)(3) + (-2y+x)(2w)(2r) + (-2y + x)(-5)(3)

now is that the final answer? or should everything be in terms of r?
 

Attachments

  • upload_2015-11-6_2-28-14.png
    upload_2015-11-6_2-28-14.png
    1.3 KB · Views: 411
  • upload_2015-11-6_2-32-55.png
    upload_2015-11-6_2-32-55.png
    1.3 KB · Views: 414
Physics news on Phys.org
I haven't checked your final differentiations, but if they are correct then the answer will be equally correct regardless of the variables in terms of which it is expressed. It may however be more conventional to express it in terms of u and r, since they are the 'ultimate' parameters. It comes down to what you think your teacher will want. If you don't know then it would be safer to express it in terms of u and r.
 
  • Like
Likes catch22
Question: A clock's minute hand has length 4 and its hour hand has length 3. What is the distance between the tips at the moment when it is increasing most rapidly?(Putnam Exam Question) Answer: Making assumption that both the hands moves at constant angular velocities, the answer is ## \sqrt{7} .## But don't you think this assumption is somewhat doubtful and wrong?

Similar threads

  • · Replies 18 ·
Replies
18
Views
3K
Replies
1
Views
1K
  • · Replies 1 ·
Replies
1
Views
1K
  • · Replies 8 ·
Replies
8
Views
2K
Replies
2
Views
2K
  • · Replies 7 ·
Replies
7
Views
3K
  • · Replies 1 ·
Replies
1
Views
2K
  • · Replies 14 ·
Replies
14
Views
3K
Replies
2
Views
2K
  • · Replies 2 ·
Replies
2
Views
3K